El arquitecto Gómez, dirige el proyecto de remodelación del parque municipal del distrito La Esperanza. La forma del parque está representada por la ecuación polar r(5-3sensθ)=16. El arquitecto planea construir un camino que une los extremos de la parte más ancha del terreno y necesita saber la distancia que existe entre los extremos (considerar que las medidas están en cientos de metros), además en el centro del camino colocará una pileta. Por ello, se requiere obtener las coordenadas de los extremos y del centro en coordenadas rectangulares. Para ayudar al arquitecto Gómez a lograr su objetivo, se deberá seguir la siguiente estrategia:
Pasar la ecuación polar a rectangular (en su forma ordinaria) (2 Puntos)
Hallar el centro, los vértices de la parte más ancha del terreno en la forma rectangular y determinar la distancia entre los vértices (considerar que las medidas están en cientos de metros), utilizando la ecuación cartesiana, hallada en a). (2 Puntos)
Graficar la cónica en el plano cartesiano ubicando las coordenadas de los vértices y del centro. (1 Punto)

Answers

Answer 1

Answer:

thank you for the point too mucheee

You: Your welcome


Related Questions

is this a direct variation

y=2x + 3

pls give an explanation if you don’t have one still pls give an answer

Answers

Answer:

No.

Step-by-step explanation:

y/x has to be the same number no matter what except at point (0 0) which it must also include for it to be a direct variation.

*y=2x+3 is not a direct variation because you can not write it as y/x=k where k is some constant number. If we were y=2x, then yes since y/x=2.

*You could also take two points and see if they are proportional. That is, you can see if y2/x2 gives the same value as y1/x1 where (x1,y1) and (x2,y2) are points on the line y=2x+3. This must work for every pair of points on the linear relation except at x=0 (where you would or should have y=0 if it is directly proportional).

Let's try it out. If x=1, then y=2(1)+3=5.

5/1=5

If x=2, then y=2(2)+3=7

7/2=3.5

As you can see 5 doesn't equal 3.5.

*For it to be a direct variation, it also must contain the point (0,0) and be a diagonal line when graphed. It can also be written in form y=kx where k is a constant number. This fails two of the the things I mentioned. It doesn't contain point (0,0) because y=2(0)+3=3 not 0. It cannot be written in form y=kx because of the plus 3.

If it were y=2x, then the answer would be yes.

3. Express the strength of a solution both as a ratio and as a percentage if
2 L of the solution contain 400 mg of solute.

Answers

Answer:

1 : 5000

0.02%

Step-by-step explanation:

A solution = solute + solvent

A 2 Litre solution = (2 * 1000) = 2000 mg

Having, 400 mg of solute ;

Recall ;

1 mg = 0.001 ml

400 mg = (0.001 * 400) = 0.4 ml

The strength of the solution :

Amount of solute / Amount of solution

0.4 / 2000

As a ratio :

0.4 / 2000 = (0.4 * 10) / (2000*10) = 4 / 20000 = 1 / 5000 = 1 : 5000 (as a ratio)

0.4 / 2000

= 0.0002

(0.0002 * 100%) = 0.02% (As a percentage)

X S2.0.2
A rocket is fired upward with an initial velocity v of 80 meters per second. The quadratic function S(t) = -52 + 80t can be used to find
the heights of the rocket, in meters, at any time t in seconds. Find the height of the rocket 8 seconds after it takes off. During the
course of its flight, after how many seconds will the rocket be at a height of 290 meters?

Answers

9514 1404 393

Answer:

320 m after 8 seconds5.6 seconds, 10.4 seconds to height of 290 m

Step-by-step explanation:

To find the height at 8 seconds, evaluate the formula for t=8.

  S(t) = -5t^2 +80t

  S(8) = -5(8^2) +80(8) = -320 +640 = 320

The height of the rocket is 320 meters 8 seconds after takeoff.

__

To find the time to 290 meters height, solve ...

  S(t) = 290

  290 = -5t^2 +80t

  -58 = t^2 -16t . . . . . . . divide by -5

  6 = t^2 -16t +64 . . . . . complete the square by adding 64

  ±√6 = t -8 . . . . . . . . . take the square root

  t = 8 ±√6 ≈ {5.551, 10.449}

The rocket is at 290 meters height after 5.6 seconds and again after 10.4 seconds.

Perimeter of a square with side 4 square root of 5

Answers

Answer:

16[tex]\sqrt{5}[/tex]

Step-by-step explanation:

[tex]4\sqrt{5}[/tex]+[tex]4\sqrt{5}[/tex]+[tex]4\sqrt{5}[/tex]+[tex]4\sqrt{5}[/tex]

16[tex]\sqrt{5}[/tex]

The perimeter of the square is 16√5 units.

We have,

The concept used here is straightforward: to find the perimeter of a square, you sum the lengths of all four sides because all sides of a square are equal in length.

In this case, the side length is given as 4√5, so you multiply it by 4 to calculate the total perimeter.

To find the perimeter (P) of a square with a side length of 4√5 units, you simply add up all four sides of the square, as all sides of a square are equal in length.

So,

P = 4 * side length

P = 4 * 4√5

P = 16√5 units

Thus,

The perimeter of the square is 16√5 units.

Learn more about squares here:

https://brainly.com/question/22964077

#SPJ3

Find the distance between the two points.
(3,-9) and (-93,-37)

Answers

Answer:

d(A,B)=100

Step-by-step explanation:

The distance between two points A([tex]x_A,y_A[/tex]) and B=([tex]x_B,y_B[/tex]) is:

d(A,B)=[tex]\sqrt{(x_B-x_A)^2+(y_B-y_A)^2}[/tex]

In this case:

[tex]x_B = - 93\\x_A = 3\\y_B = -37\\y_A = -9[/tex]

In this case:

[tex]d(A,B)=\sqrt{( - 93 -3)^2+( - 37 - (-9))^2} =\\=\sqrt{( - 96)^2+(-28)^2} =\\=\sqrt{9.216+784} \\=\sqrt{10000}=\\=\sqrt{10^4} =\\=100[/tex]

Think about tossing two coins.
What is
P (H on first coin)? ………………………….
P (H on second coin)? ……………………..
List the paired outcomes for tossing two coins: ………………………………
How many ways are there for two coins to land? ………………………
What is P (HH)? …………………………​

Answers

Given:

Two coins are tossed.

To find:

1. P(H on first coin)?

2. P(H on second coin)?

3. List the paired outcomes for tossing two coins.

4. How many ways are there for two coins to land?

5. What is P(HH)?

Solution:

If a a coin is tossed, then we have to possible outcomes, i.e., heads (H) and tails (T).

It is given that two coins are tossed.

1. The probability of getting a heads on first coin is:

[tex]P(H \text{ on first coin})=\dfrac{1}{2}[/tex]

2. The probability of getting a heads on second coin is:

[tex]P(H \text{ on second coin})=\dfrac{1}{2}[/tex]

3. If two coins are tossed, then the total possible outcomes are:

[tex]\{HH,HT,TH,TT\}[/tex]

4. The number of ways for two coins to land is 4.

5. The probability of the heads on both tosses is:

[tex]P(HH)=\dfrac{1}{4}[/tex]

Therefore, the required solution are:

1. [tex]P(H \text{ on first coin})=\dfrac{1}{2}[/tex]

2. [tex]P(H \text{ on second coin})=\dfrac{1}{2}[/tex]

3. List of possible outcomes is [tex]\{HH,HT,TH,TT\}[/tex].

4. Number of possible outcomes is 4.

5. [tex]P(HH)=\dfrac{1}{4}[/tex]

It has been determined that 60% of the people in a certain midwest city who are responsible for preparing the evening meal have no idea what they are going to prepare as late as 4PM in the afternoon. A recent survey was conducted from 1000 of these individuals. For the sampling distribution of the sample proportion to be reasonably Normal, the sample must have been obtained in the right way (ideally, a simple random sample) and the sample size must be large (so that at least 10 or more successes and failures). Are these conditions met

Answers

Answer:

Random sample, [tex]np \geq 10[/tex] and [tex]n(1-p) \geq 10[/tex], so yes, both conditions were satisfied.

Step-by-step explanation:

60% of the people in a certain midwest city who are responsible for preparing the evening meal have no idea what they are going to prepare as late as 4PM in the afternoon.

This means that [tex]p = 0.6[/tex]

A recent survey was conducted from 1000 of these individuals.

This means that [tex]n = 1000[/tex]

Also, a random sample, so the first condition was satisfied.

The sample size must be large (so that at least 10 or more successes and failures).

[tex]np = 1000*0.6 = 600 \geq 10[/tex]

[tex]n(1-p) = 1000*0.4 = 400 \geq 10[/tex]

So yes, both conditions were met.

Parallel lines

What is the segment

Answers

Answer:

Step-by-step explanation:

A segment means: each of parts into which something is or may be divided.

Instructions: Drag and drop the correct name for each angle. Each angle has more than one name so be sure to identity all the correct names

Answers

Answer/Step-by-step explanation:

Recall: an angle can be named in three different ways:

i. Using one letter which is the vertex of the angle. i.e. if the vertex of the angle is A we can name the angle as <A.

ii. Using the number of the labelled angle. i.e. is the angle is labelled 2, we can name it <2

iii. Using the three letters of the angles with the vertex angle in the middle. i.e. if the three points that form an angle are A, B, C and the vertex is B, we can name the angle as <ABC.

✔️Let's name the each angle given according:

1. <G, <3, and <FGH

2. <D, <4, and <CDE

3. <S and <TSR (the number seems blur and difficult to read. Whatever number is used to label the angle is what you'd use in naming the angle)

Solve the given system by the substitution method.
3x + y = 14
7x - 4y = 20​

Answers

Answer:

(4, 2 )

Step-by-step explanation:

Given the 2 equations

3x + y = 14 → (1)

7x - 4y = 20 → (2)

Rearrange (1) making y the subject by subtracting 3x from both sides

y = 14 - 3x → (3)

Substitute y = 14 - 3x into (2)

7x - 4(14 - 3x) = 20 ← distribute parenthesis and simplify left side

7x - 56 + 12x = 20

19x - 56 = 20 ( add 56 to both sides )

19x = 76 ( divide both sides by 19 )

x = 4

Substitute x = 4 into (3) for corresponding value of y

y = 14 - 3(4) = 14 - 12 = 2

solution is (4, 2 )

Answer:

[tex]3x + y = 14 \\ y = 14 - 3x \\ substitute \: y \: into \: equation \: 2\\ 7x - 4(14 - 3x) = 20 \\ 7x - 56 + 12x = 20 \\ 19x = 76 \\ x = \frac{76}{19} =4 \\ y = 14 - 3( 4 ) = 2 \\ [/tex]

A venture capital company feels that the rate of return (X) on a proposed investment is approximately normally distributed with mean 30% and standard deviation 10%.
(a) Find the probability that the return will exceed 55%.
(b) Find the probability that the return will be less than 25%
(c) What is the expected value of the return?
(d) Find the 75th percentile of returns.

Answers

Answer:

a) 0.0062 = 0.62% probability that the return will exceed 55%.

b) 0.3085 = 30.85% probability that the return will be less than 25%

c) 30%.

d) The 75th percentile of returns is 36.75%.

Step-by-step explanation:

Normal Probability Distribution

Problems of normal distributions can be solved using the z-score formula.

In a set with mean [tex]\mu[/tex] and standard deviation [tex]\sigma[/tex], the z-score of a measure X is given by:

[tex]Z = \frac{X - \mu}{\sigma}[/tex]

The Z-score measures how many standard deviations the measure is from the mean. After finding the Z-score, we look at the z-score table and find the p-value associated with this z-score. This p-value is the probability that the value of the measure is smaller than X, that is, the percentile of X. Subtracting 1 by the p-value, we get the probability that the value of the measure is greater than X.

Mean 30% and standard deviation 10%.

This means that [tex]\mu = 30, \sigma = 10[/tex]

(a) Find the probability that the return will exceed 55%.

This is 1 subtracted by the p-value of Z when X = 55. So

[tex]Z = \frac{X - \mu}{\sigma}[/tex]

[tex]Z = \frac{55 - 30}{10}[/tex]

[tex]Z = 2.5[/tex]

[tex]Z = 2.5[/tex] has a p-value of 0.9938

1 - 0.9938 = 0.0062

0.0062 = 0.62% probability that the return will exceed 55%.

(b) Find the probability that the return will be less than 25%

p-value of Z when X = 25. So

[tex]Z = \frac{X - \mu}{\sigma}[/tex]

[tex]Z = \frac{25 - 30}{10}[/tex]

[tex]Z = -0.5[/tex]

[tex]Z = -0.5[/tex] has a p-value of 0.3085

0.3085 = 30.85% probability that the return will be less than 25%.

(c) What is the expected value of the return?

The mean, that is, 30%.

(d) Find the 75th percentile of returns.

X when Z has a p-value of 0.75, so X when Z = 0.675.

[tex]Z = \frac{X - \mu}{\sigma}[/tex]

[tex]0.675 = \frac{X - 30}{10}[/tex]

[tex]X - 30 = 0.675*10[/tex]

[tex]X = 36.75[/tex]

The 75th percentile of returns is 36.75%.

me to
ICS A
V
t
V
30
A vehicle accelerates from 0 to 30 m/s in 10 seconds on a
straight road, then travels 15 seconds at a constant velocity.
Next it slows down, coming to a stop in 5 seconds. The car
waits 10 seconds, and then backs up for 5 seconds
accelerating from 0 to -10 m/s. Draw a graph showing the
vehicle's velocity vs time by following these steps.
20
What is the velocity of the vehicle at 0 seconds?
v m/s
Velocity (m/s)
es
10
40
20 30
Time (s)
Elementary
-10
S
Secondary
< Previous Activity
enuity.com/ContentViewers/Frame Chain/Activity
US 1:09

Answers

Hshshshgsvsvevrgeuuee

Ellen, Nick, and Ryan went shopping together. One of them bought a hat, another bought sunglasses, and another bought a belt. One paid $6, another paid $8, and another paid $10.
1) Nick bought the hat.
2) Ellen spent $8.
3) The belt did not cost $10.
4) Ryan spent the most. Which of the following is true?

(a) Nick bought the hat for $10.
(b) Ellen bought the belt for $8.
(c) Ryan bought the sunglasses for $8.
(d) Ryan bought the belt for $10
(e) Ryan bought the hat for 56.​

Answers

Answer:

see down

Step-by-step explanation:

d is correct answer

Which are correct representations of the inequality -3(2x-5) <5(2 - x)? Select two options.

Answers

Answer:

-6x+15 < 10-5x

x>5

third equation, first graph

Step-by-step explanation:

evaluate:(0.0001)-¾

Answers

Answer:

0.001

Step-by-step explanation:

(0.0001)^-3/4=((0.1)⁴)^-3/4

(0.1)^4×-3/4

0.1^‐3

0.001

Tom's graduation picnic costs $4 for every attendee. At most how many attendees can there be if Tom budgets a total of $36 for his graduation picnic?

Answers

36 divided by $4 = 9

If Tom budgets $36 for his graduation, means that was how much that was earned.

A committee raised 7/9 of their target goal last year and another 1/9 of the goal this year. What fraction of their goal has been raised

Answers

First, find 40% of 3,000:

0.40 * 3000 = 1200

So they raised 1200 so far.

Now subtract 1200 from the total they want to raise:

3000 - 1200 = 1800

So they need $1,800 more to meet their goal.

Answer:

8iriiruruj

Step-by-step explanation:

u4u the UK. We have a good idea to advertise the UK. I have been a

B
13 ft.
5 ft.
A
C
12 ft.
Find the value of Cos (B) =

Answers

Answer: the answer is 12/13

A, B and C are collinear points. B is between A and C. AB=12 BC=18 AC=3x Find X.

Answers

Answer:

[tex]x =10[/tex]

Step-by-step explanation:

Given

[tex]AB = 12[/tex]

[tex]BC = 18[/tex]

[tex]AC = 3x[/tex]

Required

Solve for x

Since B is in between both points, then:

[tex]AC = AB + BC[/tex]

This gives

[tex]3x = 12 + 18[/tex]

[tex]3x = 30[/tex]

Divide by 3

[tex]x =10[/tex]

The diameter of one circle is represented by 12x. The diameter of another circle is represented by 6x2y what is the ratio of the radiu of the two circles. 2:x3y 2x:y x:2y 2:xy

Answers

9514 1404 393

Answer:

  (d)  2 : xy

Step-by-step explanation:

A common factor of 6x can be removed from the elements of the ratio. The ratio of radii is the same as the ratio of diameters.

  12x : 6x²y = (6x)(2) : (6x)(xy) = 2 : xy

Answer:D

Step-by-step explanation:

convert the fraction 3/8 to a decimal WITHOUT the use of a calculator. Show your method clearly. SHOW ALL STEPS!

Answers

here you go it's too easy

Step-by-step explanation:

Explanation is in the attachment .

Hope it is helpful to you ❣️☪️❇️

The distribution of the annual incomes of a group of middle management employees approximated a normal distribution with a mean of $38,000 and a standard deviation of $1,000. About 68 percent of the incomes lie between what two incomes

Answers

Answer:

68% is a special

value for these problems

empirical rule suggests ± 1 standard deviation

z = (x - μ)/σ

1 = (x - 38000)/1000

Between $37,000 and $39,000

Step-by-step explanation:

The number 55 is attached to a two-digit number to its left and the formed 4-digit number is divisible by 24. What could be the 2-digit number? List all options.

Answers

Given:

The number 55 is attached to a two-digit number to its left and the formed 4-digit number is divisible by 24.

To find:

The 2-digit numbers.

Solution:

Let the two digit number be [tex]ab[/tex]. Then the 4 digit number will be [tex]55ab[/tex].

We know that the number [tex]55ab[/tex] lies in the range 5500 to 5599.

Now,

[tex]5500=229\times 24+4[/tex]

It means, [tex]5500-4=5496[/tex] is divisible by 24. So, the numbers lie in the range 5500 to 5599 and divisible by 24 are:

[tex]5496+24=5520[/tex]

[tex]5520+24=5544[/tex]

[tex]5544+24=5568[/tex]

[tex]5568+24=5592[/tex]

Therefore, the possible 2-digit numbers are 20, 44, 68, 92.

for a science fair project javier is recording the amount of water that evaporate from a bucket in a month he creates a table like this i will give point for the best answer

week 1 2/16 inch

week 2 1/16 inch


week 3 3/16 inch

week 4 2/16 inch

how much water had evaported from the bucket at the end of week 2

what was the total amount of water that evaported in the four weeks

if javier orignally put 4 inches of water in the bucket how many inches of water were left after the experment was completed

Answers

Answer: [tex]\dfrac{3}{16},\ \dfrac{1}{2}, \dfrac{7}{2}\ \text{inch}[/tex]

Step-by-step explanation:

Given

Javier created a table for the amount of water evaporated in each week

After two weeks, the amount of water evaporated is

[tex]\Rightarrow \dfrac{2}{16}+\dfrac{1}{16}\\\\\Rightarrow \dfrac{2+1}{16}=\dfrac{3}{16}\ \text{inch}[/tex]

Total amount of water evaporated in four weeks is

[tex]\Rightarrow \dfrac{2}{16}+\dfrac{1}{16}+\dfrac{3}{16}+\dfrac{2}{16}\\\\\Rightarrow \dfrac{2+1+3+2}{16}=\dfrac{8}{16}\\\Rightarrow \dfrac{1}{2}\ \text{inch}[/tex]

If Javier originally puts 4 inches of water, amount of water left in the bucket

[tex]\Rightarrow 4-\dfrac{1}{2}\\\\\Rightarrow \dfrac{4\times 2}{2}-\dfrac{1}{2}\\\\\Rightarrow \dfrac{8-1}{2}=\dfrac{7}{2}\ \text{inch}[/tex]

Find the perimeter of a football field which measures 90m by 60m

Answers

Hello!

[tex]\large\boxed{P = 300m}[/tex]

Use the following formula for the perimeter:

P = 2l + 2w, where:

l = length

w = width

Therefore:

P = 2(90) + 2(60)

Simplify:

P = 180 + 120 = 300 m

Answer:

well how about you use common sense 100 yards long on each side 200 yards then add 5o yards since the the that is how wide it is then add another 50 and you get 300 yards then convert that to meters

2.What is the value of x if x/4 + 12 = 4 ?​

Answers

Answer:

Step-by-step explanation:

Answer:

hope it will help u

Help me find the domain and range please!

Answers

Answer:

Domain: (-∞, 1]

Range: (-∞, 3]

Step-by-step explanation:

The function starts at point (1, 3) and goes to the left and down forever.

Domain: (-∞, 1]

Range: (-∞, 3]

Answer:

Domain: [tex](-\infty, 1][/tex]

Range: [tex](-\infty, 3][/tex]

Step-by-step explanation:

The domain of a function represents the range of x-values that are part of the function, read left to right. We can see that the function goes forever to the left and stops at [tex]x=1[/tex] when we read left to right. Therefore, the domain of this function is [tex]\boxed{(-\infty, 1]}[/tex].

The point at [tex]x=1[/tex] is a filled-in solid dot so it is included as part of the function. Use square brackets to denote inclusive.

The range of a function represents all y-values that are part of the function, read bottom to top. The function continues down forever and stops at [tex]y=3[/tex] when read bottom to top. Therefore, the range of this function is [tex]\boxed{(-\infty, 3]}[/tex]. Similar to the domain, we use a square bracket on the right to indicate that [tex]y=3[/tex] is included in the function. If the dot was not filled-in, then we would use a parenthesis to indicate that [tex]y=3[/tex] would not be part of the function.

Can someone please help?

Answers

Step-by-step explanation:

So, so, to attempt this, we need to use the formula :-

2 (l + b) × h ---> For Lateral surface area

2(30+30) h = 7200

2×60×h = 7200

120 × h = 7200

h = 7200/120

h = 60 cm

Now, volume = l×b×h

= 30×30×60

= 54000 cm³ is the required answer.

Hope it helps! :D

A bottle maker believes that 23% of his bottles are defective. If the bottle maker is accurate, what is the probability that the proportion of defective bottles in a sample of 602 bottles would differ from the population proportion by less than 4%

Answers

Answer:

0.9802 = 98.02% probability that the proportion of defective bottles in a sample of 602 bottles would differ from the population proportion by less than 4%

Step-by-step explanation:

To solve this question, we need to understand the normal probability distribution and the central limit theorem.

Normal Probability Distribution

Problems of normal distributions can be solved using the z-score formula.

In a set with mean [tex]\mu[/tex] and standard deviation [tex]\sigma[/tex], the z-score of a measure X is given by:

[tex]Z = \frac{X - \mu}{\sigma}[/tex]

The Z-score measures how many standard deviations the measure is from the mean. After finding the Z-score, we look at the z-score table and find the p-value associated with this z-score. This p-value is the probability that the value of the measure is smaller than X, that is, the percentile of X. Subtracting 1 by the p-value, we get the probability that the value of the measure is greater than X.

Central Limit Theorem

The Central Limit Theorem establishes that, for a normally distributed random variable X, with mean [tex]\mu[/tex] and standard deviation [tex]\sigma[/tex], the sampling distribution of the sample means with size n can be approximated to a normal distribution with mean [tex]\mu[/tex] and standard deviation [tex]s = \frac{\sigma}{\sqrt{n}}[/tex].

For a skewed variable, the Central Limit Theorem can also be applied, as long as n is at least 30.

For a proportion p in a sample of size n, the sampling distribution of the sample proportion will be approximately normal with mean [tex]\mu = p[/tex] and standard deviation [tex]s = \sqrt{\frac{p(1-p)}{n}}[/tex]

A bottle maker believes that 23% of his bottles are defective.

This means that [tex]p = 0.23[/tex]

Sample of 602 bottles

This means that [tex]n = 602[/tex]

Mean and standard deviation:

[tex]\mu = p = 0.23[/tex]

[tex]s = \sqrt{\frac{p(1-p)}{n}} = \sqrt{\frac{0.23*0.77}{602}} = 0.0172[/tex]

What is the probability that the proportion of defective bottles in a sample of 602 bottles would differ from the population proportion by less than 4%?

p-value of Z when X = 0.23 + 0.04 = 0.27 subtracted by the p-value of Z when X = 0.23 - 0.04 = 0.19.

X = 0.27

[tex]Z = \frac{X - \mu}{\sigma}[/tex]

By the Central Limit Theorem

[tex]Z = \frac{X - \mu}{s}[/tex]

[tex]Z = \frac{0.27 - 0.23}{0.0172}[/tex]

[tex]Z = 2.33[/tex]

[tex]Z = 2.33[/tex] has a p-value of 0.9901

X = 0.19

[tex]Z = \frac{X - \mu}{s}[/tex]

[tex]Z = \frac{0.19 - 0.23}{0.0172}[/tex]

[tex]Z = -2.33[/tex]

[tex]Z = -2.33[/tex] has a p-value of 0.0099

0.9901 - 0.0099 = 0.9802

0.9802 = 98.02% probability that the proportion of defective bottles in a sample of 602 bottles would differ from the population proportion by less than 4%

The length of a rectangle is 6 inches more than the width. The perimeter is 28 inches. Find the length and the width (in inches).

Answers

Answer:

The length of the rectangle is 10 inches, and the width is 4 inches.

Step-by-step explanation:

Given that the length of a rectangle is 6 inches more than the width, and the perimeter is 28 inches, the following calculation must be performed to find the length and the width:

(X + X + 6) x 2 = 28

2X + 2X + 12 = 28

4X = 28 - 12

X = 16/4

X = 4

Therefore, the length of the rectangle is 10 inches, and the width is 4 inches.

Other Questions
how do you solve for y? help solve for x pt2. The distance between Ali's house and 1 pointcollege is exactly 135 miles. If shedrove 2/3 of the distance in 135minutes. What was her average speedin miles per hour? how many distinct permutations can be formed using the letters of the word robberies What's bigger 3.05kg or 3.56 4. Which equations have 4 as a possible solution for p? Select all that apply. A. p= 8 B.p= 8 c.p= 16 D.p= 64 E.p=64 190 of 76 7 8 9 10-3456The slope of the line shown in the graph isand the intercept of the line is Mt cng ty sn xut vn trt c th bn mt ci vn trt vi gi $60. Tng chi ph cho sn xut bao gm chi ph c nh l $1200 v chi ph sn xut mt ci vn trt l $35.Nu cng ty bn c 80 ci vn trt th cng ty A rectangle has a length of 7 in. and a width of 2 in. if the rectangle is enlarged using a scale factor of 1.5, what will be the perimeter of the new rectangle How to solve an inequality for example 4x the number of multiples of a given number is infinite ( ) Unlike sociologists, the knowledge that most people possess about the world can be described as:_______ a. academic. b. systemic, clear, and consistent. c. practical. d. scientific In Circle H with m 8(x-10)=4x+40 8(x-10)=4x+40 According to the Vaporization Heat table, the heat needed for 1 mol of H2O to evaporate at 100C is 40.7KJ and 44.0KJ/mol is needed to evaporate H2O at 25C. Thus 44.0-40.7=3.7KJ is the energy needed to heat H2O to 100C from 25C.However, according to the heat capacity of H2O, 3.7KJ will only warm the water by ~+43C, which is not enough to reach 100C starting from 25C!Am I missing something?! A 15kg mass suspended from a ceiling is pulled aside with a horizontal force, F. Calculate the value of the tension. 5x-22 3x +105 x minus 22 3 X + 10 Which of the following is NOT a key research skill?A. Finding a topic where there is a lot of material you can use to research.B. Identifying a key question or problemC. Assessing the usefulness of each sourceD. Presenting your findings in a clear, organized way. grafique la recta que pasa por el punto (-2,4) y tiene pendiente m= -2/3 PLS HELP ME!!- What is the total cost for 4.6 pounds of blackberry jam and 1.6 pounds of blueberry jam? boysenberry jam $5/1b blackberry jam $5/1b raspberry jam $7/1b apple butter $3/1b marmalade $3/1b blueberry jam $8/1b Submit I don't kn